Tải bản đầy đủ (.pdf) (80 trang)

MỘT SỐ LỚP BẤT ĐẲNG THỨC VÀ BÀI TOÁN CỰC TRỊ VỚI ĐA THỨC ĐỐI XỨNG BA BIẾN

Bạn đang xem bản rút gọn của tài liệu. Xem và tải ngay bản đầy đủ của tài liệu tại đây (498.04 KB, 80 trang )

Header Page 1 of 126.

ĐẠI HỌC QUỐC GIA HÀ NỘI
TRƯỜNG ĐẠI HỌC KHOA HỌC TỰ NHIÊN
-----------------------

NGUYỄN TÀI TUỆ

MỘT SỐ LỚP BẤT ĐẲNG THỨC
VÀ BÀI TOÁN CỰC TRỊ VỚI ĐA THỨC
ĐỐI XỨNG BA BIẾN

Chuyên ngành: PHƯƠNG PHÁP TOÁN SƠ CẤP
Mã số:
60.46.01.13

LUẬN VĂN THẠC SỸ KHOA HỌC

NGƯỜI HƯỚNG DẪN KHOA HỌC:
GS.TSKH. NGUYỄN VĂN MẬU

Hà Nội – Năm 2014

Footer Page 1 of 126.


Header Page 2 of 126.

Mục lục
MỞ ĐẦU


3

1

5
5
6
6
6
7
7

2

Một số kiến thức bổ trợ
1.1 Đa thức đối xứng ba biến . . . . . . . .
1.2 Tính chất cơ bản của bất đẳng thức . .
1.3 Bất đẳng thức thường dùng . . . . . . .
1.3.1 Bất đẳng thức AM-GM . . . . .
1.3.2 Bất đẳng thức Cauchy - Schwarz
1.3.3 Bất đẳng thức Karamata . . . .

.
.
.
.
.
.

.

.
.
.
.
.

.
.
.
.
.
.

.
.
.
.
.
.

.
.
.
.
.
.

.
.
.

.
.
.

.
.
.
.
.
.

.
.
.
.
.
.

.
.
.
.
.
.

.
.
.
.
.

.

.
.
.
.
.
.

.
.
.
.
.
.

.
.
.
.
.
.

.
.
.
.
.
.


.
.
.
.
.
.

Bất đẳng thức có tổng các biến không đổi
2.1 Bất đẳng thức có tổng các biến không đổi với hàm phân thức hữu tỉ
2.1.1 Sử dụng bất đẳng thức AM-GM . . . . . . . . . . . . . . .
2.1.2 Sử dụng bất đẳng thức Cauchy-Schwarz . . . . . . . . . . .
2.1.3 Sử dụng các tính chất của hàm số . . . . . . . . . . . . . .
2.1.4 Bài toán liên quan . . . . . . . . . . . . . . . . . . . . . . .
2.2 Bất đẳng thức có tổng các biến không đổi với hàm vô tỉ . . . . . .
2.2.1 Sử dụng bất đẳng thức AM-GM . . . . . . . . . . . . . . .
2.2.2 Sử dụng bất đẳng thức Cauchy-Schwarz . . . . . . . . . . .
2.2.3 Sử dụng các tính chất của hàm số . . . . . . . . . . . . . .
2.2.4 Bài toán liên quan . . . . . . . . . . . . . . . . . . . . . . .

9
9
9
15
21
31
33
33
36
41
43


3 Bất đẳng thức có tích các biến không đổi
3.1 Bất đẳng thức có tích các biến không đổi với hàm phân thức hữu tỉ
3.1.1 Sử dụng bất đẳng thức AM-GM . . . . . . . . . . . . . . .
3.1.2 Sử dụng bất đẳng thức Cauchy-Schwarz . . . . . . . . . . .
3.1.3 Sử dụng các tính chất của hàm số . . . . . . . . . . . . . .
3.1.4 Bài toán liên quan . . . . . . . . . . . . . . . . . . . . . . .
3.2 Bất đẳng thức có tích các biến không đổi với hàm vô tỉ . . . . . .
3.2.1 Sử dụng bất đẳng thức AM-GM . . . . . . . . . . . . . . .
3.2.2 Sử dụng bất đẳng thức Cauchy-Schwarz . . . . . . . . . . .
3.2.3 Sử dụng các tính chất của hàm số . . . . . . . . . . . . . .
3.2.4 Bài toán liên quan . . . . . . . . . . . . . . . . . . . . . . .

45
45
45
50
53
55
56
56
59
60
62

1

Footer Page 2 of 126.



Header Page 3 of 126.
MỤC LỤC

4 Một số lớp bài toán cực trị với đa thức đối
4.1 Sử dụng bất đẳng thức AM-GM . . . . . .
4.2 Sử dụng bất đẳng thức Cauchy-Schwarz . .
4.3 Sử dụng các tính chất của hàm số . . . . . .
4.4 Bài toán liên quan . . . . . . . . . . . . . . .

xứng ba
. . . . . .
. . . . . .
. . . . . .
. . . . . .

biến
. . . .
. . . .
. . . .
. . . .

.
.
.
.

.
.
.
.


.
.
.
.

63
63
68
73
77

KẾT LUẬN

78

TÀI LIỆU THAM KHẢO

79

2

Footer Page 3 of 126.


Header Page 4 of 126.

MỞ ĐẦU
Bất đẳng thức là một nội dung cổ điển và quan trọng của Toán học. Ngay
từ đầu, sự ra đời và phát triển của bất đẳng thức đã đặt dấu ấn quan trọng,

chúng có sức hút mạnh mẽ đối với những người yêu toán, không chỉ ở vẻ đẹp
hình thức mà cả những bí ẩn nó mang đến luôn thôi thúc người làm toán phải
tìm tòi, sáng tạo. Bất đẳng thức còn có nhiều ứng dụng trong các môn khoa
học khác và trong thực tế. Ngày nay, bất đẳng thức vẫn luôn chiếm một vai trò
quan trọng và vẫn thường xuất hiện trong các kì thi quốc gia, quốc tế, Olympic.
Là một giáo viên THPT, tôi muốn nghiên cứu sâu hơn về bất đẳng thức nhằm
nâng cao chuyên môn phục vụ cho quá trình giảng dạy và bồi dưỡng học sinh
giỏi, vậy nên tôi đã chọn bất đẳng thức làm luận văn thạc sĩ của mình.
Bất đẳng thức vô cùng rộng lớn, trong thời gian ngắn, tôi chỉ có thể nghiên
cứu lĩnh vực nhỏ trong đó. Dưới sự hướng dẫn của GS. TSKH Nguyễn Văn Mậu,
tác giả đã hoàn thành luận văn với để tài
"Một số lớp bất đẳng thức và bài toán cực trị với đa thức đối xứng
ba biến."
Luận văn được chia làm bốn chương:
• Chương 1: Một số kiến thức bổ trợ.
• Chương 2: Bất đẳng thức với tổng không đổi.
• Chương 3: Bất đẳng thức có tích không đổi.
• Chương 4: Một số lớp bài toán cực trị với đa thức đối xứng ba biến.

Mặc dù có nhiều cố gắng, song do thời gian và trình độ còn hạn chế nên luận
văn khó tránh khỏi những thiếu sót. Vì vậy tác giả rất mong nhận được sự góp
ý của các thầy cô và các bạn để luận văn được hoàn thiện hơn.
Qua luận văn này, tác giả xin bày tỏ lòng biết ơn sâu sắc đến GS.TSKH
Nguyễn Văn Mậu, người Thầy đã truyền cho tác giả có niềm say mê nghiên cứu
3

Footer Page 4 of 126.


Header Page 5 of 126.

MỞ ĐẦU

toán học. Thầy đã tận tình hướng dẫn, giúp đỡ tác giả trong suốt quá trình học
tập và hoàn thiện luận văn này.
Tác giả xin chân thành cảm ơn Ban giám hiệu, Phòng Đào tạo Sau đại học,
Khoa Toán- Cơ - Tin, các thầy cô đã tạo điều kiện thuận lợi cho em hoàn thành
bản luận văn này.
Em xin chân thành cảm ơn!
Hà Nội, ngày 01 tháng 12 năm 2014
Tác giả

4

Footer Page 5 of 126.


Header Page 6 of 126.

Chương 1

Một số kiến thức bổ trợ
1.1

Đa thức đối xứng ba biến

1.1.1 Các khái niệm cơ bản
Định nghĩa 1.1. Một đơn thức ϕ(x, y, z) của các biến x, y, z được hiểu là hàm
số có dạng
ϕ(x, y, z) = aklm xk y l z m ,


trong đó k, l, m ∈ N được gọi là bậc của biến x, y, z , số aklm ∈ R∗ = R\{0} được
gọi là hệ số của đơn thức, còn số k + l + m được gọi là bậc của đơn thức ϕ(x, y, z).
Định nghĩa 1.2. Một hàm số P (x, y, z) của các biến x, y, z được gọi là một đa
thức nếu nó có thể được biểu diễn ở dạng tổng hữu hạn các đơn thức
aklm xk y l z m ,

P (x, y, z) =

n ∈ N.

k,l,m∈N
k+l+m=n

Bậc lớn nhất của các đơn thức trong đa thức được gọi là bậc của đa thức.
Định nghĩa 1.3. Đa thức P (x, y, z) được gọi là đối xứng, nếu nó không thay
đổi với mọi hoán vị của x, y, z , nghĩa là
P (x, y, z) = P (y, x, z) = P (z, y, x) = P (x, z, y).

Định nghĩa 1.4. Đa thức f (x, y, z) được gọi là thuần nhất bậc m, nếu
f (tx, ty, tz) = tm f (x, y, z),

t=0

Định nghĩa 1.5. Các đa thức
σ1 = x + y + z, σ2 = xy + yz + zx, σ3 = xyz,

được gọi là đa thức đối xứng cơ sở của các biến x, y, z.
1.1.2 Tổng lũy thừa
5


Footer Page 6 of 126.


Header Page 7 of 126.
Chương 1. Một số kiến thức bổ trợ

Định nghĩa 1.6. Các đa thức sk = xk + y k + z k , (k = 0, 1, ...), được gọi là tổng
lũy thừa bậc k của các biến x, y, z.
Định lý 1.1 ( Công thức Newton). Với mọi k ∈ Z, ta có hệ thức
sk = σ1 sk−1 − σ2 sk−2 + σ3 sk−3 .

Định lý 1.2. Một tổng lũy thừa sk = xk + y k + z k đều có thể biểu diễn được
dưới dạng một đa thức bậc n theo các biến σ1 , σ2 , σ3 .
Định lý 1.3 (Công thức Waring). Tổng lũy thừa sk được biểu diễn qua cá đa
thức đối xứng cở sở theo công thức
sk
=
k

1.2

0≤l,m,n
l+2m+3n=k

(−1)k−l−m−n (l + m + n − 1)! l m n
σ1 σ2 σ3 .
l!m!n!

Tính chất cơ bản của bất đẳng thức


1. a > b ⇔ a + c > b + c.
2. a > b, b > c thì a > c.
3. a > b thì
ca > cb khi c > 0
ca < cb khi c < 0.

4. a > b, c > d thì a + c > b + d.
5. a > b > 0, c > d > 0 thì ac > bd.
6. Với n nguyên dương, ta có
a < b ⇔ a2n+1 < b2n+1
0 < a < b ⇒ a2n < b2n .

1.3

Bất đẳng thức thường dùng

1.3.1

Bất đẳng thức AM-GM

Định lý 1.4. Giả sử a1 , a2 , . . . , an là các số thực không âm, khi đó ta luôn có

a1 + a2 + · · · + an
≥ n a1 a2 . . . an .
n

Đẳng thức xảy ra khi và chỉ khi a1 = a2 = · · · = an .
6

Footer Page 7 of 126.



Header Page 8 of 126.
Chương 1. Một số kiến thức bổ trợ

Hệ quả 1.1. Với mọi số thực dương a1 , a2 , . . . , an ta có
1
1
1
+
+ ··· +
a1 a2
an

(a1 + a2 + · · · + an ) ≥ n2 .

Đẳng thức xảy ra khi và chỉ khi a1 = a2 = · · · = an .
Hệ quả 1.2. Với mọi số thực a, b, c, ta luôn có
1. a2 + b2 + c2 ≥ ab + bc + ca
2. a2 + b2 + c2 ≥

(a + b + c)2
3

3. (a + b + c)2 ≥ 3(ab + bc + ca)
4. a2 b2 + b2 c2 + c2 a2 ≥ abc(a + b + c)
5. (ab + bc + ca)2 ≥ 3abc(a + b + c).
1.3.2

Bất đẳng thức Cauchy - Schwarz


Định lý 1.5. Nếu a1 , a2 , . . . , an , b1 , b2 , . . . , bn là các số thực tùy ý thì
(a1 b1 + a2 b2 + · · · + an bn )2 ≤ a21 + a22 + · · · + a2n

Đẳng thức xảy ra khi và chỉ khi

b21 + b22 + · · · + b2n .

( )

a2
an
a1
=
= ··· =
( ở đây ta sử dụng quy ước
b1
b2
bn

nếu mẫu bằng 0 thì tử cũng bằng 0).
x
yi

Nhận xét 1.1. Theo bất đẳng thức ( ), chọn ai = √ i và bi =



yi với xi , yi ∈


R, yi > 0. Ta thu được bất đẳng thức Cauchy-Schwarz dạng phân thức ( hay còn
gọi là bất đẳng thức Cauchy-Schwarz dạng Engel).
Hệ quả 1.3. Nếu x1 , x2 , . . . , xn là các số thực và y1 , y2 , . . . , yn là các số thực
dương thì
x21 x22
x2
(x1 + x2 + . . . xn )2
+
+ ··· + n ≥
.
y1
y2
yn
y1 + y2 + · · · + yn
x
x
x
Đẳng thức xảy ra khi và chỉ khi 1 = 2 = · · · = n .
y1
y2
yn
1.3.3

Bất đẳng thức Karamata

Định lý 1.6. Cho hai dãy số {xk , yk ∈ I(a, b), k = 1, 2, . . . , n}, thỏa mãn điều kiện
x1 ≥ x2 ≥ · · · ≥ xn , y1 ≥ y2 ≥ · · · ≥ yn

7


Footer Page 8 of 126.


Header Page 9 of 126.
Chương 1. Một số kiến thức bổ trợ




x1 ≥ y1




 x1 + x2 ≥ y 1 + y 2
........



x1 + x2 + · · · + xn−1 ≥ y1 + y2 + · · · + yn−1



x 1 + x2 + · · · + xn = y 1 + y 2 + · · · + y n

Khi đó, ứng với hàm số lồi f (x)(f (x) ≥ 0) trên I(a, b), ta đều có
f (x1 ) + f (x2 ) + · · · + f (xn ) ≥ f (y1 ) + f (y2 ) + · · · + f (yn ).

Đẳng thức xảy ra khi và chỉ khi xi = yi , i = 1, 2, . . . n.
Ta cũng phát biểu tương tự đối với hàm số lõm bằng cách đổi chiều dấu bất

đẳng thức.
Bổ đề 1.1. Cho hàm số y = f (x) liên tục và có đạo hàm cấp 2 trên I(a; b).
a. Nếu f (x) ≥ 0, ∀x ∈ I(a; b) thì f (x) ≥ f (x0 )(x − x0 ) + f (x0 ), ∀x0 ∈ I(a; b).
b. Nếu f (x) ≤ 0, ∀x ∈ I(a; b) thì f (x) ≤ f (x0 )(x − x0 ) + f (x0 ), ∀x0 ∈ I(a; b).
Đẳng thức trong hai bất đẳng thức trên xảy ra khi và chỉ khi x = x0 .

8

Footer Page 9 of 126.


Header Page 10 of 126.

Chương 2

Bất đẳng thức có tổng các biến
không đổi
2.1
2.1.1

Bất đẳng thức có tổng các biến không đổi với hàm phân
thức hữu tỉ
Sử dụng bất đẳng thức AM-GM

Đối với bất đẳng thức P (x, y, z) ≥ 0 (≤ 0), Trong đó P (x, y, z) là đa thức hoặc
phân thức hữu tỉ và có tổng x + y + z không đổi, thì khi đó sử dụng các kĩ thuật
của bất đẳng thức AM − GM như dự đoán dấu bằng xảy ra, AM − GM ngược
dấu, đặt ẩn phụ, ... tỏ ra rất hiệu quả.
Bài toán 2.1. Với a, b, c là các số thực dương thỏa mãn a + b + c = 3. Chứng
minh rằng

a2
b2
c2
+
+
≥ 1.
b+2 c+2 a+2

Chứng minh.
Áp dụng bất đẳng thức AM-GM, ta có
b+2
2a
a2
+

b+2
9
3
c+2
2b
b2
+

c+2
9
3
2
c
a+2
2c

+
≥ .
a+2
9
3

Cộng các bất đẳng thức cùng chiều ta được
a2
b2
c2
5
2
+
+
≥ (a + b + c) − = 1.
b+2 c+2 a+2
9
3

Đẳng thức xảy ra khi và chỉ khi a = b = c = 1.

9

Footer Page 10 of 126.


Header Page 11 of 126.
Chương 2. Bất đẳng thức có tổng các biến không đổi

Bài toán 2.2. Với a, b, c là các số thực dương thỏa mãn a + b + c = 3. Chứng

minh rằng
2(ab + bc + ca) +

1
1
1
+ +
≥ 9.
ab bc ca

Chứng minh.
Cộng cả hai vế của bất đẳng thức cần chứng minh với a2 + b2 + c2 , ta được
1
1
1
+ +
≥ 9 + a2 + b 2 + c 2
ab bc ca

a2 + b2 + c2 + 2(ab + bc + ca) +
⇔(a + b + c)2 +

1
1
1
+ +
≥ 9 + a2 + b2 + c2
ab bc ca

1

1
1
+ +
≥ a2 + b 2 + c 2
ab bc ca
a+b+c

≥ a2 + b 2 + c 2
abc
3

≥ a2 + b 2 + c 2
abc
⇔abc(a2 + b2 + c2 ) ≤ 3



⇔abc(a + b + c)(a2 + b2 + c2 ) ≤ 9.

Ta có
abc(a + b + c)(a2 + b2 + c2 ) ≤

(ab + bc + ca)2 2
(a + b2 + c2 ).
3

Do đó ta chứng minh
(ab + bc + ca)2 2
(a + b2 + c2 ) ≤ 9 ⇔ (ab + bc + ca)2 (a2 + b2 + c2 ) ≤ 27.
3


Áp dụng bất đẳng thức AM-GM, ta có
2

2

2

(ab + bc + ca) + (ab + bc + ca) + a2 + b2 + c2
3

2

(ab + bc + ca) (a + b + c ) ≤
=

(a + b + c)2
3

3

3

= 27.

Vậy bất đẳng thức được chứng minh.
Đẳng thức xảy ra khi và chỉ khi a = b = c = 1.
Bài toán 2.3. Với các số thực không âm a, b, c thỏa mãn a + b + c = 3. Chứng
minh rằng
2 a2 b + b2 c + c2 a + 3 a2 + b2 + c2 + 4abc ≥ 19.


Chứng minh.

10

Footer Page 11 of 126.


Header Page 12 of 126.
Chương 2. Bất đẳng thức có tổng các biến không đổi

Ta có
19 = 3 (a + b + c)2 − 8 = 3 a2 + b2 + c2 + 6 (ab + bc + ca) − 8.

Khi đó bất đẳng thức cần chứng minh trở thành
2 a2 b + b2 c + c2 a + 4abc ≥ 6 (ab + bc + ca) − 8
⇔a2 b + b2 c + c2 a + 2abc + 4 ≥ 3 (ab + bc + ca) .

Mà ta lại có
3 (ab + bc + ca) = (a + b + c) (ab + bc + ca)
= a2 b + b2 c + c2 a + ab2 + bc2 + ca2 + 3abc.

Do đó bất đẳng thức cần chứng minh tương đương với
4 ≥ ab2 + bc2 + ca2 + abc.

Không mất tính tổng quát giả sử a ≥ b ≥ c ≥ 0
⇒ (b − a) (b − c) ≤ 0
⇔b2 − bc − ab + ac ≤ 0
⇔ab2 + a2 c ≤ a2 b + abc.


Như vậy
ab2 + bc2 + ca2 + abc ≤ bc2 + abc + a2 b + abc
= bc2 + 2abc + a2 b
= b (a + c)2
a+ca+c
= 4b
2
2

a + c a + c 3
+
b+
2
2 
≤ 4
3
= 4.

Vậy bất đẳng thức được chứng minh.
Đẳng thức xảy ra khi và chỉ khi a = b = c = 1.

11

Footer Page 12 of 126.


Header Page 13 of 126.
Chương 2. Bất đẳng thức có tổng các biến không đổi

Bài toán 2.4 (Bulgaria TST 2003). Với ba số thực dương a, b, c thỏa mãn điều

kiện a + b + c = 3. Chứng minh rằng
a
b
c
3
+
+
≥ .
2
2
2
1+b
1+c
1+a
2

Chứng minh.
Áp dụng bất đẳng thức AM-GM, ta có
a
ab
ab2
ab2
=a− .
=
a


a

2

2
1+b
1+b
2b
2

Tương tự, ta có
b
bc

b

1 + c2
2
c
ac
≥c− .
2
a +1
2

Cộng vế với vế, ta có
a
b
c
1
ab + bc + ca
+
+
≥ a + b + c − (ab + bc + ca) = 3 −

.
2
2
2
1+b
1+c
1+a
2
2

Mặt khác, ta có
ab + bc + ca ≤

do đó

(a + b + c)2
=3
3

a
b
c
ab + bc + ca
3
+
+
≥3−
≥ .
2
2

2
1+b
1+c
1+a
2
2

Đẳng thức xảy ra khi và chỉ khi a = b = c = 1.

Bài toán 2.5 (Turkey TST 2007). Với a, b, c là các số thực dương thỏa mãn
a + b + c = 1. Chứng minh rằng
1
1
1
1
+
+

.
2
2
2
ab + 2c + 2c bc + 2a + 2a ca + 2b + 2b
ab + bc + ca

Chứng minh.
Áp dụng bất đẳng thức AM-GM, ta có

12


Footer Page 13 of 126.


Header Page 14 of 126.
Chương 2. Bất đẳng thức có tổng các biến không đổi

ab + 2c2 + 2c = ab + 2c2 + 2c(a + b + c)
= ab + 4c2 + 2ac + 2bc
= (b + 2c)(a + 2c)
a(b + 2c)b(a + 2c)
=
ab
(ab + 2ac)(ab + 2bc)
=
ab
[ab + 2ac + ab + 2bc]2

4ab
(ab + bc + ca)2
=
ab

từ đó suy ra
ab
1

.
2
ab + 2c + 2c
(ab + bc + ca)2


Tương tự, ta có
1
bc

2
bc + 2a + 2a
(ab + bc + ca)2
ca
1

.
2
ca + 2b + 2b
(ab + bc + ca)2

Cộng các bất đẳng thức cùng chiều, ta có
1
1
1
1
+
+

.
2
2
2
ab + 2c + 2c bc + 2a + 2a ca + 2b + 2b
ab + bc + ca

1
3

Đẳng thức xảy ra khi và chỉ khi a = b = c = .
Bài toán 2.6. Với các số thực dương a, b, c thỏa mãn a + b + c = 3. Chứng minh
rằng
a2 + bc b2 + ca c2 + ab
+
+
≥ 3.
b + ca
c + ab
a + bc

Chứng minh.
Bất đẳng thức cần chứng minh tương đương với
a2 + bc
b2 + ca
c2 + ab
+
+
≥1
3b + 3ca 3c + 3ab 3a + 3bc
a2 + bc
b2 + ca
c2 + ab

+
+
≥ 1.

b(a + b + c) + 3ca c(a + b + c) + 3ab a(a + b + c) + 3bc

Mặt khác
3b + 3ac = b(a + b + c) + 3ac ≤ b(a + b + c) + ac + a2 + c2 = a2 + b2 + c2 + ab + bc + ca
13

Footer Page 14 of 126.


Header Page 15 of 126.
Chương 2. Bất đẳng thức có tổng các biến không đổi

do đó

a2 + bc
a2 + bc
≥ 2
.
3b + 3ac
a + b2 + c2 + ab + bc + ca

Tương tự, ta có
b2 + ca
b2 + ca
≥ 2
3c + 3ab
a + b2 + c2 + ab + bc + ca
c2 + ab
c2 + ab
≥ 2

.
3a + 3bc
a + b2 + c2 + ab + bc + ca

Cộng các bất đẳng thức cùng chiều, ta có
a2 + bc
b2 + ca
c2 + ab
+
+
≥ 1.
3b + 3ca 3c + 3ab 3a + 3bc

Bất đẳng thức được chứng minh.
Đẳng thức xảy ra khi và chỉ khi a = b = c = 1.

14

Footer Page 15 of 126.


Header Page 16 of 126.
Chương 2. Bất đẳng thức có tổng các biến không đổi

2.1.2

Sử dụng bất đẳng thức Cauchy-Schwarz

Nếu yêu cầu của bất đẳng thức là chứng minh P (x, y, z) ≥ 0(≤ 0) với P (x, y, z)
có dạng tổng các bình phương hoặc phân thức với tử số của mỗi phân thức có

dạng bình phương như vậy ta có thể nghĩ đến việc áp dụng bất đẳng thức
Cauchy- Schwarz. Hoặc sử dụng giả thiết x + y + z không đổi ta có thể biến đổi
để đưa bất đẳng thức về dạng trên để sử dụng Cauchy-Schwarz.
Bài toán 2.7. Với các số thực dương a, b, c thỏa mãn điều kiện a + b + c = 1.
Chứng minh rằng
a
b
c
+
+
≥ 1.
1+b−a 1+c−b 1+a−c

Chứng minh.
Ta luôn có
b
c
a
b
c
a
+
+
=
+
+
1+b−a 1+c−b 1+a−c
2b + c 2c + a 2a + b
a2
b2

c2
=
+
+
.
2ab + ac 2bc + ab 2ac + bc

Áp dụng bất đẳng thức Cauchy-Schwarz, ta có
a2
b2
c2
(a + b + c)2
+
+

.
2ab + ac 2bc + ab 2ac + bc
3 (ab + bc + ca)

Theo hệ quả của bất đẳng thức AM-GM, ta có
(a + b + c)2 ≥ 3(ab + bc + ca)

do đó

b2
c2
(a + b + c)2
a2
+
+


≥ 1.
2ab + ac 2bc + ab 2ac + bc
3 (ab + bc + ca)

Đẳng thức xảy ra khi và chỉ khi a = b = c = 1.
Bài toán 2.8. Với các số thực dương a, b, c sao cho a + b + c = 1. Chứng minh
a2

1
1
1
1
+
+ +
≥ 30.
2
2
+b +c
ab bc ca

Chứng minh.
Áp dụng bất đẳng thức Cauchy-Schwarz cho hai bộ số


1
a2

+ b2


+ c2

1
1
1
,√ ,√ ,√
ab bc cd
15

Footer Page 16 of 126.


Header Page 17 of 126.
Chương 2. Bất đẳng thức có tổng các biến không đổi






a2 + b2 + c2 , 3 ab, 3 bc, 3 ca

ta có
1
1
1
1
+
+
a2 + b2 + c2 + 9ab + 9bc + 9ca ≥ (1 + 3 + 3 + 3)2

+
a2 + b2 + c2 ab bc ca
1
1
1
1
⇔ 2
+ +
(a + b + c)2 + 7(ab + bc + ca) ≥ 100
+
2
2
a +b +c
ab bc ca
1
1
1
100
1
⇔ 2
+ +

.
+
2
2
2
a +b +c
ab bc ca
(a + b + c) + 7(ab + bc + ca)


Theo hệ quả của bất đẳng thức AM − GM, ta có
(a + b + c)2
≥ ab + bc + ca
3

nên
a2

1
1
1
1
+
+ +

2
2
+b +c
ab bc ca

100
(a + b + c)2
1+7
3
= 30.
1
3

Đẳng thức xảy ra khi và chỉ khi a = b = c = .


Bài toán 2.9 (Iran MO TST 2009). Với các số thực dương a, b, c thỏa mãn điều
kiện a + b + c = 3. Chứng minh rằng
a2

1
1
1
3
+ 2
+ 2
≤ .
2
2
2
+b +2 b +c +2 c +a +2
4

Chứng minh.
Bất đẳng thức cần chứng minh
1
1
1
1
1
1
3 3
− 2
+
− 2

+
− 2
≥ −
2
2
2
2 a +b +2
2 b +c +2
2 c +a +2
2 4
2
2
2
2
2
2
a +b
b +c
c +a
3
⇔ 2
+ 2
+ 2

2
2
2
a +b +2 b +c +2 c +a +2
2
2

2
2
2
(a + b) + (a − b)
(b + c) + (b − c)
(c + a)2 + (c − a)2

+
+
≥3
a2 + b2 + 2
b2 + c 2 + 2
c 2 + a2 + 2
(a + b)2
(b + c)2
(c + a)2
⇔ 2
+
+
a + b2 + 2 b2 + c2 + 2 c2 + a2 + 2


+

(a − b)2
(b − c)2
(c − a)2
+
+
≥ 3.

a2 + b 2 + 2 b 2 + c 2 + 2 c 2 + a2 + 2

16

Footer Page 17 of 126.


Header Page 18 of 126.
Chương 2. Bất đẳng thức có tổng các biến không đổi

Theo bất đẳng thức Cauchy-Schwarz, ta có
(b + c)2
(c + a)2
[2(a + b + c)]2
(a + b)2
+
+

a2 + b 2 + 2 b 2 + c 2 + 2 c 2 + a2 + 2
2a2 + 2b2 + 2c2 + 6

(1)

(b − a)2
(b − c)2
(c − a)2
(2b − 2a)2
+
+


.
a2 + b 2 + 2 b 2 + c 2 + 2 c 2 + a2 + 2
2a2 + 2b2 + 2c2 + 6

(2)



Bất đẳng thức được chứng minh nếu
4 (a + b + c)2 + (2b − 2a)2
≥3
2a2 + 2b2 + 2c2
⇔2(a + b + c)2 + 2(b − a) ≥ 3(a2 + b2 + c2 + 3)
⇔ a2 + b 2 + c 2

2

+ 2 (b − a)2 ≥ a2 + b2 + c2

⇔bc − ab + ca − c2 ≥ 0
⇔(c − a)(b − c) ≥ 0.

(3)

Có thể nhận thấy bất đẳng thức (3) không phải luôn đúng nhưng ta có thể
ép nó đúng. Thật vậy sử dụng các đánh giá tương tự (3), ta có
(a − b)2
(c − b)2
(c − a)2
(2c − 2b)2

+
+

a2 + b2 + 2 b2 + c2 + 2 c2 + a2 + 2
2a2 + 2b2 + 2c2 + 6
(b − c)2
(a − c)2
(2a − 2c)2
(a − b)2
+
+

a2 + b2 + 2 b2 + c2 + 2 c2 + a2 + 2
2a2 + 2b2 + 2c2 + 6

và cùng lần lượt đưa bài toán về chứng minh
(a − b)(c − a) ≥ 0

(4)

(b − c)(a − b) ≥ 0.

(5)

Như vậy nếu trong các bất đẳng thức (3),(4) và (5) có một bất đẳng thức
đúng thì bài toán sẽ được chứng minh song. Ta thấy rằng
[(c − a)(b − c)] [(a − b)(c − a)] [(b − c)(a − b)] = (a − b)2 (b − c)2 (c − a)2 ≥ 0

nên ít nhất một trong ba số (c-a)(b-c), (a-b)(c-a),(b-c)(a-b) sẽ có một số không
âm. Tức là phải có ít nhất một bất đẳng thức đúng hay bất đẳng thức được

chứng minh.
Đẳng thức xảy ra khi và chỉ khi a = b = c = 1.

17

Footer Page 18 of 126.


Header Page 19 of 126.
Chương 2. Bất đẳng thức có tổng các biến không đổi

Nhận xét 2.1. Có nhiều công cụ hỗ trợ ta thực hiện phương pháp dồn biến,
dưới đây ta xem xét ứng dụng yếu tố "ít nhất" và bất đẳng thức Cauchy-Schwarz
trong việc giảm biến số của bất đẳng thức. Cụ thể có thể đưa bất đẳng thức ba
biến về bất đẳng thức một biến để chứng minh. Ý tưởng của kĩ thuật như sau:
Với bốn số thực a, b, c, k ta có
[(a − k)(b − k)] [(b − k)(c − k)] [(c − k)(a − k)] = (a − k)2 (b − k)2 (c − k)2 ≥ 0.

Do đó trong ba số (a − k)(b − k), (b − k)(c − k), (c − k)(a − k) sẽ có "ít nhất"
một số không âm. Giả sử (a − k)(b − k) ≥ 0 thế thì
a2 + b2 = k 2 + (a + b − k)2 − 2(a − k)(b − k) ≤ k 2 + (a + b − k)2 .

Như vậy để chứng minh bất đẳng thức có giả thiết dạng a + b + c = s và đẳng
thức xảy ra khi 2 biến đằng một giá trị nào đó ta có thể sử dụng đánh giá trên
để làm giảm biến số của bất đẳng thức ban đầu. Cụ thể chọn k = m ( đảm bảo
dấu bằng) và ta có
a2 + b2 ≤ m2 + (a + b − m)2 = m2 + (s − c − m)2 .

Nghĩa là có thể sử dụng đánh giá này vào bài toán thì ta sẽ chỉ còn phải
chứng minh bất đẳng thức của một biến c.

Bài toán 2.10. Với a, b, c là các số thực thỏa mãn điều kiện a + b + c = 3. Chứng
minh rằng
7a2

a
b
c
1
+ 2
+ 2
≤ .
+ 11 7b + 11 7c + 11
6

Chứng minh.
Bất đẳng thức cần chứng minh
14a
14b
14c
14
+1− 2
+1− 2
≥3−
+ 11
7b + 11
7c + 11
6
2
2
2

7(a − 1) + 4 7(b − 1) + 4 7(c − 1) + 4
2

+
+

2
2
2
7a + 11
7b + 11
7c + 11
3
2
1
(a − 1)
1
1
(b − 1)2
(c − 1)2
2
⇔4
+
+
+
7
+
+
≥ .
2

2
2
2
2
2
7a + 11 7b + 11 7c + 11
7a + 11 7b + 11 7c + 11
3
⇔1 −

7a2

Áp dụng bất đẳng thức Cauchy -Schwarz, ta có
1
1
1
(1 + 1 + 1)2
+
+

7a2 + 11 7b2 + 11 7c2 + 11
7(a2 + b2 + c2 ) + 33
9
=
2
2
7(a + b + c2 ) + 33
3(a + b + c)2
=
21(a2 + b2 + c2 ) + 11(a + b + c)2

18

Footer Page 19 of 126.


Header Page 20 of 126.
Chương 2. Bất đẳng thức có tổng các biến không đổi

và cũng theo bất đẳng thức Cauchy-Schwarz, ta có
(1 − a)2
(b − 1)2
(c − 1)2
(1 − a + b − 1 + c − 1)2
+
+

7a2 + 11 7b2 + 11 7c2 + 11
7(a+ b2 + c2 ) + 33
(b + c − a − 1)2
=
7(a2 + b2 + c2 ) + 11.3
2

a+b+c
3 b+c−a−
3
=
2
2
2

21(a + b + c ) + 11.(a + b + c)2
4(b + c − 2a)2
=
.
3 [21(a2 + b2 + c2 ) + 11.(a + b + c)2 ]

Bất đẳng thức được chứng minh nếu
4.3(a + b + c)2
7.4(b + c − 2a)2
2
+

2
2
2
2
2
2
2
2
21(a + b + c ) + 11(a + b + c)
3 [21(a + b + c ) + 11.(a + b + c) ]
3
2
2
2
2
2
2
⇔2.9(a + b + c) + 7.2(b + c − 2a) ≥ 21(a + b + c ) + 11(a + b + c)

⇔42 [a(a − c) − b(a − c)] ≥ 0
⇔42(a − c)(a − b) ≥ 0.

(1)

Rõ ràng (1 ) không phải lúc nào cũng đúng, nhưng từ đó vẫn đi đến điều
phải chứng minh bằng cách đánh giá tương tự
(1 − b)2
(c − 1)2
4(a + c − 2b)2
(a − 1)2
+
+

7a2 + 11 7b2 + 11 7c2 + 11
3 [21(a2 + b2 + c2 ) + 11.(a + b + c)2 ]
(a − 1)2
(b − 1)2
(1 − c)2
4(a + b − 2c)2
+
+

7a2 + 11 7b2 + 11 7c2 + 11
3 [21(a2 + b2 + c2 ) + 11.(a + b + c)2 ]

và ta lần lượt đưa bài toán về xét tính đúng sai của
42(b − a)(b − c) ≥ 0

(2)


42(c − a)(c − b) ≥ 0

(3)

mà ta lại có
(a − b)(a − c) + (b − a)(b − c) + (c − a)(c − b) = a2 + b2 + c2 − ab − bc − ca ≥ 0.

Do đó trong (1),(2),(3) có ít nhất một bất đẳng thức đúng. Từ đó suy ra điều
phải chứng minh.
Đẳng thức xảy ra khi và chỉ khi a = b = c = 1.

19

Footer Page 20 of 126.


Header Page 21 of 126.
Chương 2. Bất đẳng thức có tổng các biến không đổi

Bài toán 2.11. Chứng minh rằng với các số thực dương a, b, c tùy ý ta đều có
(b + c − a)2
(c + a − b)2
(a + b − c)2
3
+
+
≥ .
2
2

2
2
2
2
a + (b + c)
b + (c + a)
c + (a + b)
5

Chứng minh.
Do bất đẳng thức đã cho thuần nhất với biến a, b, c nên ta hoàn toàn có thể
chuẩn hóa cho a + b + c = 1. Khi đó bất đẳng thức đã cho được viết dưới dạng
(1 − 2a)2
(1 − 2b)2
(1 − 2c)2
3
+
+

.
2a2 − 2a + 1 2b2 − 2b + 1 2c2 − 2c + 1
5

Ta có
1
3

a−
1
= a−

3

1
3
1
b−
3
b−

2

b−
2

1
c−
3

1
3

c−

a2 + b 2 ≤

c−

1
3


a−

1
3

2

≥ 0.

Không mất tính tổng quát, giả sử a −
Ta có

1
3

1
3

b−

1
3

≥ 0.

1
1
1
2
+ (a + b − )2 = +

−c
9
3
9
3

2

.

Áp dụng bất đẳng thức Cauchy-Schwarz, ta có
(1 − 2a)2
(1 − 2b)2
[2 − 2(a + b)]2
+

2a2 − 2a + 1 2b2 − 2b + 1
2(a2 + b2 ) − 2(a + b) + 2
[2 − 2(a + b)]2

2
1
+ ( − c)2 − 2 (1 − c) + 2
2
9
3
2c2
=
2
1

2
+
−c +c
9
3
18c2
= 2
.
9c − 3c + 5

Bất đẳng thức được chứng minh nếu ta chứng minh được
(1 − 2c)2
18c2
+
≥ 35
2c2 − 2c + 1 9c2 − 3c + 5
⇔(3c − 1)2 (17c2 − 8c + 5) ≥ 0

Đẳng thức xảy ra khi và chỉ khi a = b = c.

20

Footer Page 21 of 126.

luôn đúng .


Header Page 22 of 126.
Chương 2. Bất đẳng thức có tổng các biến không đổi


2.1.3

Sử dụng các tính chất của hàm số

Trong các bài toán chứng minh bất đẳng thức dạng P (x, y, z) ≥ 0(≤ 0), dựa
vào tổng không đổi và dựa vào dự đoán dấu đẳng thức xảy ra khi nào. Ta có
thể đưa bài toán về dạng hàm một biến và sử dụng các tính chất của hàm số để
khảo sát. Hoặc sử dụng bất đẳng thức Karamata và bổ đề cơ bản.
Bài toán 2.12. Với x, y, z > 0 thỏa mãn x + y + z = 1. Chứng minh rằng
1
1
+
≥ 16.
xz yz

Chứng minh.
Yêu cầu bài toán ⇔

1
z

1 1
+
x y

≥ 16.

Theo bất đẳng thức AM-GM, ta có
1 1
4

4
+ ≥
=
.
x y
x+y
1−z

Ta đi chứng minh

1 4
.
≥ 16, z ∈ (0; 1).
z 1−z
1 4
4
Xét hàm số f (z) = .
− 16 =
− 16, với z ∈ (0; 1)
2
z 1−z
−z + z
−4(−2z + 1)
.
(−z 2 + z)2
1
f (z) = 0 ⇔ z = .
2
f (z) =


Ta lập bảng biến thiên
z

1
2

0


f (z)
f (z)

0

1
+

+∞

+∞
0

Từ bảng biến thiên, ta có f (z) ≥ 0, với ∀z ∈ (0; 1).
1
4

1
2

Đẳng thức xảy ra khi và chỉ khi x = y = , z = .


Bài toán 2.13 (Tiệp Khắc MO ,1984). Với x, y, z ≥ 0 thỏa mãn x + y + z = 1.
Chứng minh rằng
xy + yz + zx − 2xyz ≤
21

Footer Page 22 of 126.

7
.
27


Header Page 23 of 126.
Chương 2. Bất đẳng thức có tổng các biến không đổi

Chứng minh.
Đặt xy + yz + zx − 2xyz = P .
Giả sử
x = min{x, y, z}.
1
3

Từ giả thiết x, y, z ≥ 0, x + y + z = 1 ⇒ 0 ≤ x ≤ .
Theo bất đẳng thức AM-GM, ta có
(1 − x)2

1 − x = y + z ≥ 2 yz ⇒
≥ yz.
4


Do đó
P = xy + yz + zx − 2xyz
= (xy + zx) + (yz − 2xyz)
= x(y + z) + yz(1 − 2x)
(1 − x)2
(1 − 2x)
4
−2x3 + x2 + 1
=
= f (x).
4
≤ x(1 − x) +

Xét hàm số f (x) =

−2x3 + x2 + 1
1
với x ∈ 0; ,
4
3
−6x2 + 2x
4
x=0
1 .
⇒f (x) = 0 ⇔
x=
3
f (x) =


Ta có bảng biến thiên
x

0

f (x)

0

f (a)

1
4

1
3

+

0
7
27

Từ bảng biên thiên ta thấy với ∀x ∈ 0;

1
, ta có
3

P ≤ f (x) ≤


7
.
27
1
3

Đẳng thức xảy ra khi và chỉ khi x = y = z = .

22

Footer Page 23 of 126.


Header Page 24 of 126.
Chương 2. Bất đẳng thức có tổng các biến không đổi

Bài toán 2.14. Với x, y, z ≥ 0 thỏa mãn x + y + z = 1. Chứng minh rằng
xy + yz + zx ≤

1 9
+ xyz.
4 4

Chứng minh.
1
Giả sử z = min{x, y, z} ⇒ 0 ≤ z ≤ .
3

Có x + y + z = 1 ⇒ 1 − z = x + y.

Theo bất đẳng thức AM-GM, ta có
(x + y)2
≥ xy.
4
9
4

1
4

Bất đẳng thức cần chứng minh tương đương với xy + yz + zx − xyz ≤ .
Ta có
9
9
xy + yz + zx − xyz = xy + z(x + y) − xyz
4
4
9
= xy(1 − z) + z(x + y)
4
(x + y)2
9
(1 − z) + z(1 − z)

4
4
2
(1 − z)
9z
=

(1 − ) + z − z 2
4
4
1
= (−9z 3 + 6z 2 − z + 4).
16
1

Xét hàm f (z) = −9z 3 + 6z 2 − z + 4, với z ∈ 1; .
3

1
3
Ta có f (z) = −27z 2 + 12z − 1 ⇒ f (z) = 0 ⇔ 
1
z= .
9
z=

Ta lập bảng biến thiên
z

1
9

0

f (z) | −
4
f (z)


0

1
3

+

320
81

Từ bảng biến thiên ta thấy f (z) ≤ 4, ∀z ∈ 0;
Vậy xy + yz + zx ≤

1 9
+ xyz.
4 4

0
4
1
.
3

1
z = 0, x = y =
2
Đẳng thức xảy ra khi và chỉ khi 
1
x=y=z= .

3



23

Footer Page 24 of 126.


Header Page 25 of 126.
Chương 2. Bất đẳng thức có tổng các biến không đổi

Bài toán 2.15. Với x, y, z > 0 và x + y + z = 1. Chứng minh rằng
5
2(x3 + y 3 + z 3 ) + 3(x2 + y 2 + z 2 ) + 12xyz ≥ .
3

Chứng minh.
1
Giả sử x = min{x, y, z} ⇒ x ∈ 0; .
3

Từ giả thiết ta có y + z = 1 − x và theo bất đẳng thức AM-GM, ta có
yz ≤

(y + z)2
.
4

Đặt 2(x3 + y 3 + z 3 ) + 3(x2 + y 2 + z 2 ) + 12xyz = P.

Khi đó, ta có
P = 2 x3 + (y + z)3 − 3yz(y + z) + 3 x2 + (y + z)2 − 2yz + 12xyz
= 2x3 + 2(1 − x)3 − 6yz(1 − x) + 3x2 + 3(1 − x)2 − 6yz + 12xyz
= 12x2 − 12x + 5 − 6yz(2 − 3x)
≥ 12x2 − 12x + 5 − 6

(y + z)2
(2 − 3x)
4

9
3
= x3 − x + 2.
2
2
9
2

3
1
.
2
3
3
1
27
Ta có f (x) = x2 − ⇒ f (x) = 0 ⇔ x = .
2
2
3


Đặt x3 − x + 2 = f (x) với x ∈ 0;

Ta có bảng biến thiên
x
f (x)
f (x)

0

1
3

| −
2

0
5
3

Từ bảng biến thiên ta suy ra
P ≥ f (x) ≥

5
1
với x ∈ 0; .
3
3
1
3


Đẳng thức xảy ra khi và chỉ khi x = y = z = .

Bài toán 2.16. Với x, y, z ∈ [0; 2] thỏa mãn điều kiện x + y + z = 3. Chứng minh
rằng
x3 + y 3 + z 3 ≤ 9.
24

Footer Page 25 of 126.


×